LSAT and Law School Admissions Forum

Get expert LSAT preparation and law school admissions advice from PowerScore Test Preparation.

User avatar
 Dave Killoran
PowerScore Staff
  • PowerScore Staff
  • Posts: 5852
  • Joined: Mar 25, 2011
|
#59731
Complete Question Explanation
(The complete setup for this game can be found here: lsat/viewtopic.php?t=6078)

The correct answer choice is (A)

The Local condition in the question stem produces the following mini-diagram:

pt1_j91_g1_q2.png
Since P and N are a block, when L sits next to P, either an LPN or NPL block is formed (the diagram above has LPN). Since L must sit next to N or M, and N is already occupied, it follows that M must sit next to L. There are now only two open spaces. Since K cannot sit next to M, K must sit next to N. O sits in the final chair, next to K and M. Answer choice (A) is correct.
You do not have the required permissions to view the files attached to this post.

Get the most out of your LSAT Prep Plus subscription.

Analyze and track your performance with our Testing and Analytics Package.